find a general formula for $E(X^t)$ when X has the log-normal distribution












1














Finding the answer to this integral is looking trivial to me.



I have to start with that:



$$E(X^t)=int_0^infty x^t frac
{1}{sqrt {2pi}x}e^{-ln(x)^2/2}dx$$



I am basically stuck on this integral: with $ln x=y$



$$frac{1}{sqrt {2 pi}}int_0^infty e^{yt} e^{-y^2/2}dy$$



I know that the answer is: $$e^{frac{t^2}{2}}$$










share|cite|improve this question




















  • 2




    Let $y=ln x-t$.
    – Jimmy R.
    Nov 29 '18 at 2:29










  • After the variable change the lower bound of the integral becomes $-infty$ and not $0$.
    – Jimmy R.
    Nov 29 '18 at 2:40
















1














Finding the answer to this integral is looking trivial to me.



I have to start with that:



$$E(X^t)=int_0^infty x^t frac
{1}{sqrt {2pi}x}e^{-ln(x)^2/2}dx$$



I am basically stuck on this integral: with $ln x=y$



$$frac{1}{sqrt {2 pi}}int_0^infty e^{yt} e^{-y^2/2}dy$$



I know that the answer is: $$e^{frac{t^2}{2}}$$










share|cite|improve this question




















  • 2




    Let $y=ln x-t$.
    – Jimmy R.
    Nov 29 '18 at 2:29










  • After the variable change the lower bound of the integral becomes $-infty$ and not $0$.
    – Jimmy R.
    Nov 29 '18 at 2:40














1












1








1







Finding the answer to this integral is looking trivial to me.



I have to start with that:



$$E(X^t)=int_0^infty x^t frac
{1}{sqrt {2pi}x}e^{-ln(x)^2/2}dx$$



I am basically stuck on this integral: with $ln x=y$



$$frac{1}{sqrt {2 pi}}int_0^infty e^{yt} e^{-y^2/2}dy$$



I know that the answer is: $$e^{frac{t^2}{2}}$$










share|cite|improve this question















Finding the answer to this integral is looking trivial to me.



I have to start with that:



$$E(X^t)=int_0^infty x^t frac
{1}{sqrt {2pi}x}e^{-ln(x)^2/2}dx$$



I am basically stuck on this integral: with $ln x=y$



$$frac{1}{sqrt {2 pi}}int_0^infty e^{yt} e^{-y^2/2}dy$$



I know that the answer is: $$e^{frac{t^2}{2}}$$







probability probability-distributions expected-value






share|cite|improve this question















share|cite|improve this question













share|cite|improve this question




share|cite|improve this question








edited Nov 29 '18 at 2:28









Jimmy R.

33k42157




33k42157










asked Nov 29 '18 at 2:19









Mahamad A. Kanouté

312111




312111








  • 2




    Let $y=ln x-t$.
    – Jimmy R.
    Nov 29 '18 at 2:29










  • After the variable change the lower bound of the integral becomes $-infty$ and not $0$.
    – Jimmy R.
    Nov 29 '18 at 2:40














  • 2




    Let $y=ln x-t$.
    – Jimmy R.
    Nov 29 '18 at 2:29










  • After the variable change the lower bound of the integral becomes $-infty$ and not $0$.
    – Jimmy R.
    Nov 29 '18 at 2:40








2




2




Let $y=ln x-t$.
– Jimmy R.
Nov 29 '18 at 2:29




Let $y=ln x-t$.
– Jimmy R.
Nov 29 '18 at 2:29












After the variable change the lower bound of the integral becomes $-infty$ and not $0$.
– Jimmy R.
Nov 29 '18 at 2:40




After the variable change the lower bound of the integral becomes $-infty$ and not $0$.
– Jimmy R.
Nov 29 '18 at 2:40










2 Answers
2






active

oldest

votes


















1














Hint:$$e^{yt}e^{-y^2/2}=e^{-frac12(y^2-2yt+t^2)+frac12t^2}=e^{t^2/2}e^{-frac12(y-t)^2}$$






share|cite|improve this answer





























    1














    (I believe you've made two small mistakes - in the second integral it should be $dy$ instead of $dx$, and the integral should be taken over $-infty$ to $infty$. Be careful when doing substitutions!)



    The trick here is in completing the square:



    $$
    begin{align}
    e^{yt} e^{-y^2/2} &= e^{-frac{1}{2} (y^2 - 2yt)} \
    &=e^{-frac{1}{2} (y^2 - 2yt+ t^2)}e^{frac{1}{2}t^2}\
    &=e^{-frac{1}{2} (y-t)^2}e^{frac{1}{2}t^2}\
    end{align}
    $$

    Putting this expression back into the original integral that you were stuck on, you'll get:
    $$
    begin{align}
    int_{-infty}^infty frac{1}{sqrt{2pi}} e^{-frac{1}{2} (y-t)^2}e^{frac{1}{2}t^2}dy
    &=e^{frac{1}{2}t^2}int_{-infty}^infty frac{1}{sqrt{2pi}} e^{-frac{1}{2} (y-t)^2}dy
    end{align}
    $$

    Observing that what you have in the integral is the PDF of the Gaussian distribution, which integrates out to 1, you will get what you want:
    $$
    begin{align}
    e^{frac{1}{2}t^2}int_{-infty}^infty frac{1}{sqrt{2pi}} e^{-frac{1}{2} (y-t)^2}dy
    end{align} = e^{frac{1}{2}t^2}
    $$






    share|cite|improve this answer





















    • you're right about the bounds. I completely forgot about it! However, my dx and dy are correctly placed
      – Mahamad A. Kanouté
      Nov 29 '18 at 3:10











    Your Answer





    StackExchange.ifUsing("editor", function () {
    return StackExchange.using("mathjaxEditing", function () {
    StackExchange.MarkdownEditor.creationCallbacks.add(function (editor, postfix) {
    StackExchange.mathjaxEditing.prepareWmdForMathJax(editor, postfix, [["$", "$"], ["\\(","\\)"]]);
    });
    });
    }, "mathjax-editing");

    StackExchange.ready(function() {
    var channelOptions = {
    tags: "".split(" "),
    id: "69"
    };
    initTagRenderer("".split(" "), "".split(" "), channelOptions);

    StackExchange.using("externalEditor", function() {
    // Have to fire editor after snippets, if snippets enabled
    if (StackExchange.settings.snippets.snippetsEnabled) {
    StackExchange.using("snippets", function() {
    createEditor();
    });
    }
    else {
    createEditor();
    }
    });

    function createEditor() {
    StackExchange.prepareEditor({
    heartbeatType: 'answer',
    autoActivateHeartbeat: false,
    convertImagesToLinks: true,
    noModals: true,
    showLowRepImageUploadWarning: true,
    reputationToPostImages: 10,
    bindNavPrevention: true,
    postfix: "",
    imageUploader: {
    brandingHtml: "Powered by u003ca class="icon-imgur-white" href="https://imgur.com/"u003eu003c/au003e",
    contentPolicyHtml: "User contributions licensed under u003ca href="https://creativecommons.org/licenses/by-sa/3.0/"u003ecc by-sa 3.0 with attribution requiredu003c/au003e u003ca href="https://stackoverflow.com/legal/content-policy"u003e(content policy)u003c/au003e",
    allowUrls: true
    },
    noCode: true, onDemand: true,
    discardSelector: ".discard-answer"
    ,immediatelyShowMarkdownHelp:true
    });


    }
    });














    draft saved

    draft discarded


















    StackExchange.ready(
    function () {
    StackExchange.openid.initPostLogin('.new-post-login', 'https%3a%2f%2fmath.stackexchange.com%2fquestions%2f3018068%2ffind-a-general-formula-for-ext-when-x-has-the-log-normal-distribution%23new-answer', 'question_page');
    }
    );

    Post as a guest















    Required, but never shown

























    2 Answers
    2






    active

    oldest

    votes








    2 Answers
    2






    active

    oldest

    votes









    active

    oldest

    votes






    active

    oldest

    votes









    1














    Hint:$$e^{yt}e^{-y^2/2}=e^{-frac12(y^2-2yt+t^2)+frac12t^2}=e^{t^2/2}e^{-frac12(y-t)^2}$$






    share|cite|improve this answer


























      1














      Hint:$$e^{yt}e^{-y^2/2}=e^{-frac12(y^2-2yt+t^2)+frac12t^2}=e^{t^2/2}e^{-frac12(y-t)^2}$$






      share|cite|improve this answer
























        1












        1








        1






        Hint:$$e^{yt}e^{-y^2/2}=e^{-frac12(y^2-2yt+t^2)+frac12t^2}=e^{t^2/2}e^{-frac12(y-t)^2}$$






        share|cite|improve this answer












        Hint:$$e^{yt}e^{-y^2/2}=e^{-frac12(y^2-2yt+t^2)+frac12t^2}=e^{t^2/2}e^{-frac12(y-t)^2}$$







        share|cite|improve this answer












        share|cite|improve this answer



        share|cite|improve this answer










        answered Nov 29 '18 at 2:31









        Jimmy R.

        33k42157




        33k42157























            1














            (I believe you've made two small mistakes - in the second integral it should be $dy$ instead of $dx$, and the integral should be taken over $-infty$ to $infty$. Be careful when doing substitutions!)



            The trick here is in completing the square:



            $$
            begin{align}
            e^{yt} e^{-y^2/2} &= e^{-frac{1}{2} (y^2 - 2yt)} \
            &=e^{-frac{1}{2} (y^2 - 2yt+ t^2)}e^{frac{1}{2}t^2}\
            &=e^{-frac{1}{2} (y-t)^2}e^{frac{1}{2}t^2}\
            end{align}
            $$

            Putting this expression back into the original integral that you were stuck on, you'll get:
            $$
            begin{align}
            int_{-infty}^infty frac{1}{sqrt{2pi}} e^{-frac{1}{2} (y-t)^2}e^{frac{1}{2}t^2}dy
            &=e^{frac{1}{2}t^2}int_{-infty}^infty frac{1}{sqrt{2pi}} e^{-frac{1}{2} (y-t)^2}dy
            end{align}
            $$

            Observing that what you have in the integral is the PDF of the Gaussian distribution, which integrates out to 1, you will get what you want:
            $$
            begin{align}
            e^{frac{1}{2}t^2}int_{-infty}^infty frac{1}{sqrt{2pi}} e^{-frac{1}{2} (y-t)^2}dy
            end{align} = e^{frac{1}{2}t^2}
            $$






            share|cite|improve this answer





















            • you're right about the bounds. I completely forgot about it! However, my dx and dy are correctly placed
              – Mahamad A. Kanouté
              Nov 29 '18 at 3:10
















            1














            (I believe you've made two small mistakes - in the second integral it should be $dy$ instead of $dx$, and the integral should be taken over $-infty$ to $infty$. Be careful when doing substitutions!)



            The trick here is in completing the square:



            $$
            begin{align}
            e^{yt} e^{-y^2/2} &= e^{-frac{1}{2} (y^2 - 2yt)} \
            &=e^{-frac{1}{2} (y^2 - 2yt+ t^2)}e^{frac{1}{2}t^2}\
            &=e^{-frac{1}{2} (y-t)^2}e^{frac{1}{2}t^2}\
            end{align}
            $$

            Putting this expression back into the original integral that you were stuck on, you'll get:
            $$
            begin{align}
            int_{-infty}^infty frac{1}{sqrt{2pi}} e^{-frac{1}{2} (y-t)^2}e^{frac{1}{2}t^2}dy
            &=e^{frac{1}{2}t^2}int_{-infty}^infty frac{1}{sqrt{2pi}} e^{-frac{1}{2} (y-t)^2}dy
            end{align}
            $$

            Observing that what you have in the integral is the PDF of the Gaussian distribution, which integrates out to 1, you will get what you want:
            $$
            begin{align}
            e^{frac{1}{2}t^2}int_{-infty}^infty frac{1}{sqrt{2pi}} e^{-frac{1}{2} (y-t)^2}dy
            end{align} = e^{frac{1}{2}t^2}
            $$






            share|cite|improve this answer





















            • you're right about the bounds. I completely forgot about it! However, my dx and dy are correctly placed
              – Mahamad A. Kanouté
              Nov 29 '18 at 3:10














            1












            1








            1






            (I believe you've made two small mistakes - in the second integral it should be $dy$ instead of $dx$, and the integral should be taken over $-infty$ to $infty$. Be careful when doing substitutions!)



            The trick here is in completing the square:



            $$
            begin{align}
            e^{yt} e^{-y^2/2} &= e^{-frac{1}{2} (y^2 - 2yt)} \
            &=e^{-frac{1}{2} (y^2 - 2yt+ t^2)}e^{frac{1}{2}t^2}\
            &=e^{-frac{1}{2} (y-t)^2}e^{frac{1}{2}t^2}\
            end{align}
            $$

            Putting this expression back into the original integral that you were stuck on, you'll get:
            $$
            begin{align}
            int_{-infty}^infty frac{1}{sqrt{2pi}} e^{-frac{1}{2} (y-t)^2}e^{frac{1}{2}t^2}dy
            &=e^{frac{1}{2}t^2}int_{-infty}^infty frac{1}{sqrt{2pi}} e^{-frac{1}{2} (y-t)^2}dy
            end{align}
            $$

            Observing that what you have in the integral is the PDF of the Gaussian distribution, which integrates out to 1, you will get what you want:
            $$
            begin{align}
            e^{frac{1}{2}t^2}int_{-infty}^infty frac{1}{sqrt{2pi}} e^{-frac{1}{2} (y-t)^2}dy
            end{align} = e^{frac{1}{2}t^2}
            $$






            share|cite|improve this answer












            (I believe you've made two small mistakes - in the second integral it should be $dy$ instead of $dx$, and the integral should be taken over $-infty$ to $infty$. Be careful when doing substitutions!)



            The trick here is in completing the square:



            $$
            begin{align}
            e^{yt} e^{-y^2/2} &= e^{-frac{1}{2} (y^2 - 2yt)} \
            &=e^{-frac{1}{2} (y^2 - 2yt+ t^2)}e^{frac{1}{2}t^2}\
            &=e^{-frac{1}{2} (y-t)^2}e^{frac{1}{2}t^2}\
            end{align}
            $$

            Putting this expression back into the original integral that you were stuck on, you'll get:
            $$
            begin{align}
            int_{-infty}^infty frac{1}{sqrt{2pi}} e^{-frac{1}{2} (y-t)^2}e^{frac{1}{2}t^2}dy
            &=e^{frac{1}{2}t^2}int_{-infty}^infty frac{1}{sqrt{2pi}} e^{-frac{1}{2} (y-t)^2}dy
            end{align}
            $$

            Observing that what you have in the integral is the PDF of the Gaussian distribution, which integrates out to 1, you will get what you want:
            $$
            begin{align}
            e^{frac{1}{2}t^2}int_{-infty}^infty frac{1}{sqrt{2pi}} e^{-frac{1}{2} (y-t)^2}dy
            end{align} = e^{frac{1}{2}t^2}
            $$







            share|cite|improve this answer












            share|cite|improve this answer



            share|cite|improve this answer










            answered Nov 29 '18 at 2:38









            Sean Lee

            1578




            1578












            • you're right about the bounds. I completely forgot about it! However, my dx and dy are correctly placed
              – Mahamad A. Kanouté
              Nov 29 '18 at 3:10


















            • you're right about the bounds. I completely forgot about it! However, my dx and dy are correctly placed
              – Mahamad A. Kanouté
              Nov 29 '18 at 3:10
















            you're right about the bounds. I completely forgot about it! However, my dx and dy are correctly placed
            – Mahamad A. Kanouté
            Nov 29 '18 at 3:10




            you're right about the bounds. I completely forgot about it! However, my dx and dy are correctly placed
            – Mahamad A. Kanouté
            Nov 29 '18 at 3:10


















            draft saved

            draft discarded




















































            Thanks for contributing an answer to Mathematics Stack Exchange!


            • Please be sure to answer the question. Provide details and share your research!

            But avoid



            • Asking for help, clarification, or responding to other answers.

            • Making statements based on opinion; back them up with references or personal experience.


            Use MathJax to format equations. MathJax reference.


            To learn more, see our tips on writing great answers.





            Some of your past answers have not been well-received, and you're in danger of being blocked from answering.


            Please pay close attention to the following guidance:


            • Please be sure to answer the question. Provide details and share your research!

            But avoid



            • Asking for help, clarification, or responding to other answers.

            • Making statements based on opinion; back them up with references or personal experience.


            To learn more, see our tips on writing great answers.




            draft saved


            draft discarded














            StackExchange.ready(
            function () {
            StackExchange.openid.initPostLogin('.new-post-login', 'https%3a%2f%2fmath.stackexchange.com%2fquestions%2f3018068%2ffind-a-general-formula-for-ext-when-x-has-the-log-normal-distribution%23new-answer', 'question_page');
            }
            );

            Post as a guest















            Required, but never shown





















































            Required, but never shown














            Required, but never shown












            Required, but never shown







            Required, but never shown

































            Required, but never shown














            Required, but never shown












            Required, but never shown







            Required, but never shown







            Popular posts from this blog

            Le Mesnil-Réaume

            Ida-Boy-Ed-Garten

            web3.py web3.isConnected() returns false always